Last visit was: 23 Apr 2024, 12:12 It is currently 23 Apr 2024, 12:12

Close
GMAT Club Daily Prep
Thank you for using the timer - this advanced tool can estimate your performance and suggest more practice questions. We have subscribed you to Daily Prep Questions via email.

Customized
for You

we will pick new questions that match your level based on your Timer History

Track
Your Progress

every week, we’ll send you an estimated GMAT score based on your performance

Practice
Pays

we will pick new questions that match your level based on your Timer History
Not interested in getting valuable practice questions and articles delivered to your email? No problem, unsubscribe here.
Close
Request Expert Reply
Confirm Cancel
SORT BY:
Kudos
Tags:
Difficulty: 555-605 Levelx   Assumptionx               
Show Tags
Hide Tags
Math Expert
Joined: 02 Sep 2009
Posts: 92881
Own Kudos [?]: 618574 [212]
Given Kudos: 81562
Send PM
Most Helpful Reply
Tutor
Joined: 16 Oct 2010
Posts: 14816
Own Kudos [?]: 64880 [31]
Given Kudos: 426
Location: Pune, India
Send PM
Manager
Manager
Joined: 15 Nov 2017
Status:Current Student
Posts: 238
Own Kudos [?]: 368 [12]
Given Kudos: 28
Concentration: Operations, Marketing
WE:Operations (Retail)
Send PM
e-GMAT Representative
Joined: 02 Nov 2011
Posts: 4341
Own Kudos [?]: 30775 [5]
Given Kudos: 632
GMAT Date: 08-19-2020
Send PM
Re: Editorial: Our city's public transportation agency is facing a budget [#permalink]
4
Kudos
1
Bookmarks
Expert Reply
The correct answer is option D.

Let us see why.

Conclusion: It is in the best interest of employees for their union to accept cuts in Retirement Benefits

Given that:
1. The transport agency is facing a budget deficit
2. Employee Retirement Benefits:
- Fastest Growing part of the budget
- Exceptionally generous (opinion of the author/editorial)
3. If the Budget shortfall is not resolved, many employees will lose their jobs

Falsification Question: in what scenario would it not be be in the best interest of employees to allow a cut in Retirement Benefits, given the above?

Falsification Condition:

There seems to be a logical jump made by the author here -

What we know about the retirement benefit:
- Fastest Growing part of the budget
- Exceptionally generous (opinion of the author/editorial)

Does the above 2 points necessarily mean that reducing the benefit will help resolve the budget deficit?

So, what we do not know - is whether reducing the retirement benefit will be able to resolve the budget deficit problem.

If it really will help resolve the budget shortfall problem, then the editorial can say that it is in the best interest of the employees to allow such a budget cut in their retirement benefit.

Assumption: Reducing the retirement benefit will be able to resolve the budget deficit issue


Option Choice Analysis:

(A) The transportation employees' union should not accept cuts in retirement benefits if doing so would not be in the employees' best interest.
Irrelevant to the argument at hand. if the cut is not in the best interest, the union should not accept it - what does this have to do with the conclusion which is about whether the cut is in the best interest of the employees or not?

(B) The only feasible way for the agency to resolve the budget shortfall would involve cutting transportation service and eliminating jobs.
Not correct. if this were true, then why is a reduction in retirement benefit even under consideration? These could be other ways to resolve the budget deficit, but the argument is whether the cut in retirement benefit is in the best interest of the employees or not.

(C) Other things being equal, it is in the transportation employees' interest to have exceptionally generous retirement benefits.
In simple words, negated option C means that a very generous retirement benefit is not in the best interest of the employees. So, cutting this would help the cause of the employees, so it will be in the best interest of the employees to allow a cut (instead of breaking the conclusion, negated option C is strengthening the conclusion). Not the correct answer.

(D) Cutting the retirement benefits would help resolve the agency's budget shortfall.
As per prethinking. if cutting the retirement benefit will not help resolve the budget shortfall, then how can it be in the best interest of the employees? (given that if the budget shortfall is not addressed, many employees will lose jobs).

(E) The transportation employees' union will not accept cuts in retirement benefits if doing so will not allow more transportation employees to keep their jobs.
Irrelevant - what the union will or will not do is not under consideration in the argument.

Hope this helps.
General Discussion
examPAL Representative
Joined: 07 Dec 2017
Posts: 1050
Own Kudos [?]: 1777 [10]
Given Kudos: 26
Send PM
Re: Editorial: Our city's public transportation agency is facing a budget [#permalink]
8
Kudos
2
Bookmarks
Javierbc wrote:
Can anybody explain why option B is not correct?


Hey Javierbc,

The negation of (B) is that 'there is some other way to fix the budget'.
But even if there is some other way, the management might still choose to cut jobs.
Since this is not a 'strict negation', i.e. the argument can still stand (if a bit weaker) without it, it is not a 'GMAT assumption'.

Negating (D), however, yields 'cutting the benefits would not fix the budget'.
Without this, the entire argument that 'workers should accept benefit cuts so as to fix the budget' is negated.
Then (D) is a much stronger choice.

Note that in general, the best way to avoid getting tripped up by confusing almost-right answers is to try identify in advance what the weakest link / correct answer likely is.
If it (or something very similar) is in your answer choices, you should feel comfortable selecting it without thinking deeply about every single one of the options.
examPAL Representative
Joined: 07 Dec 2017
Posts: 1050
Own Kudos [?]: 1777 [6]
Given Kudos: 26
Send PM
Re: Editorial: Our city's public transportation agency is facing a budget [#permalink]
6
Kudos
Bunuel wrote:
Editorial: Our city's public transportation agency is facing a budget shortfall. The fastest growing part of the budget has been employee retirement benefits, which are exceptionally generous. Unless the budget shortfall is resolved, transportation service will be cut, and many transportation employees will lose their jobs. Thus, it would be in the employees' best interest for their union to accept cuts in retirement benefits.

Which of the following is an assumption the editorial's argument requires?

A. The transportation employees' union should not accept cuts in retirement benefits if doing so would not be in the employees' best interest.
B. The only feasible way for the agency to resolve the budget shortfall would involve cutting transportation service and eliminating jobs.
C. Other things being equal, it is in the transportation employees' interest to have exceptionally generous retirement benefits.
D. Cutting the retirement benefits would help resolve the agency's budget shortfall.
E. The transportation employees' union will not accept cuts in retirement benefits if doing so will not allow more transportation employees to keep their jobs.


CR11741.01
OG2020 NEW QUESTION


The argument is basically that the workers should accept retirement benefit cuts because otherwise the budget will be insufficient and they'll lose their jobs. (This is justified by the dubious claim that these benefits are the 'fastest growing' part of the budget, which should trip your 'what?!' alarm bells).
The natural assumption behind this is that cutting these 'fastest-growing' benefits will solve the budget issues. Since we know what to look for, we'll look for it without wasting time on irrelevant options. This is a Precise approach.

(D) is exactly what we are looking for and is our answer.

Originally posted by DavidTutorexamPAL on 27 Apr 2019, 02:14.
Last edited by DavidTutorexamPAL on 03 May 2019, 12:38, edited 1 time in total.
Current Student
Joined: 10 Jun 2018
Posts: 22
Own Kudos [?]: 36 [5]
Given Kudos: 5
Location: Germany
GMAT 1: 620 Q47 V28
GMAT 2: 680 Q47 V36
GMAT 3: 710 Q49 V39
GPA: 3.14
Send PM
Re: Editorial: Our city's public transportation agency is facing a budget [#permalink]
4
Kudos
1
Bookmarks
This one is tricky. I hate assumptions questions because these answer choices that include negations always make my head spin :D Therefore I want to share my line of thinking and maybe it helps somebody :)

So the conclusion that we focus on is: It is in the employees best interest to accept cuts to their retirement benefits because otherwise they might loose their jobs through a budget shortfall.

A. The transportation employees’ union should not accept cuts in retirement benefits if doing so would not be in the employees’ best interest.

This one can be very confusing because of the negation. But looking at this answer high level what is it really saying. Its presenting a case that might be true but has nothing to do with the conclusion of the argument. "If it is not in their best interest they should not accept the cuts." We want to find something that must be assumed however that IT IS in their best interest. Therefore we can eliminate A.

B. The only feasible way for the agency to resolve the budget shortfall would involve cutting transportation service and eliminating jobs.

This is again really tricky. But it is important to recount the task. Find an assumption that is absolutely necessary. Does the cutting of services and elimination have to be the only way? No of course there could be 100 other ways and it makes no difference to our assumption. Therefore no impact on the conclusion and we can eliminate B.

C. Other things being equal, it is in the transportation employees’ interest to have exceptionally generous retirement benefits.

This one is easier to eliminate. Exceptionally is extreme language so it is definitely not necessary to assume this.

D. Cutting the retirement benefits would help resolve the agency's budget shortfall.

What if cut the retirement benefits and it would not help us resolve our main issue with the budget shortfall? This would completely destroy this argument. If we can not influence the elimination of jobs then it is not in the best interest of the employees to accept the cuts.

E. The transportation employees’ union will not accept cuts in retirement benefits if doing so will not allow more transportation employees to keep their jobs.

Again the negation that makes my head sping. But what is it saying on a high level and does it matter to the conclusion? It just says that the union will only accept if the employees really can keep their job. But this has nothing to do with the motivation of the employees to really accept those cuts.
Intern
Intern
Joined: 01 Apr 2020
Posts: 27
Own Kudos [?]: 50 [5]
Given Kudos: 6
Send PM
Re: Editorial: Our city's public transportation agency is facing a budget [#permalink]
2
Kudos
3
Bookmarks

Passage Analysis


• Editorial: Our city's public transportation agency is facing a budget shortfall.
    o The editorial states that the city’s public transportation agency is confronted by a budget shortfall.

• The fastest growing part of the budget has been employee retirement benefits, which are exceptionally generous.
    o Expenditure due to employee retirement benefits has been growing faster than all other parts of the agency’s budget..
    o The retirement benefits provided by the agency to its employees are unusually generous.

• Unless the budget shortfall is resolved, transportation service will be cut, and many transportation employees will lose their jobs.
    o If a solution is not devised for the budget shortfall, as a consequence transportation service will be terminated.
    o Loss of employment for many employees in the agency will follow if the shortfall is not resolved.

• Thus, it would be in the employees' best interest for their union to accept cuts in retirement benefits.
    o The editorial judges that if the union of workers in the agency choose to accept a cut on retirement benefits, that will be better for the employees.
    o We can infer that keeping the workers’ jobs is possibly more important in the long term.

Conclusion: It would be in the employees’ best interest for their union to accept cuts in retirement benefits.

Question Stem Analysis
We are required to identify the assumption required for the editorial’s argument to be valid.

Pre-thinking


Falsification Question
In what scenario is it possible that it would not be in the employees’ best interest for their union to accept cuts in retirement benefits?
Given that
    • The public transportation agency is facing a budget shortfall.
    • The employee retirement benefits part is the fastest growing portion of the agency’s budget.
    • The employee retirement benefits are exceptionally generous.
    • If the budget shortfall is not resolved, service will be cut, and many employees will lose their jobs.

Thought Process
The editorial suggests that it is best for the employees’ union to accept a cut in their retirement benefits because, if this step is not taken, drastic measures like cutting services will have to be followed to overcome the budget deficit. We can infer that this will lead to loss of jobs for current employees. It is a known fact that the retirement benefits for the agency’s employees are unusually generous and this part of the budget is the one growing fastest. Notice that the editorial does not say it is a large part of the budget at present. The main purpose of the measures discussed in the passage is to resolve the budget shortfall, so it will not be in the best interest of the workers if any other step that does not affect their employment could be taken instead of cutting their retirement benefits. Also, if the union agrees with the cut and this measure still does not suffice to resolve the issue (this is possible since we do not know how big the expense is currently), leading to loss of jobs for many employees, the workers are hard hit twice. This situation will not be in the workers’ best interest either.

Falsification condition#1
What if the cut in employee retirement benefits is not enough to resolve the budget shortfall and the services are cut even after that, leading to loss of employment along with the cut in retirement benefits?
In that case the facts hold true, but the conclusion breaks down.
Assumption#1
The cut in retirement benefits will lead to the resolution of the budget shortfall of the public transportation agency.

Falsification condition#2
The step of cutting employee retirement benefits will be in the best interest of the workers only when it is the only way the situation can be resolved without loss of jobs. What if it is not? Here also, the conclusion breaks down.
Assumption#2
The only method possible to resolve the budget shortfall without loss of employment of many workers is by cutting employee retirement benefits.

Answer Choice Analysis


(A) The transportation employees' union should not accept cuts in retirement benefits if doing so would not be in the employees' best interest.
INCORRECT
This statement is just the contrapositive version of the conclusion and brings no new information. Hence it cannot be an assumption

(B) The only feasible way for the agency to resolve the budget shortfall would involve cutting transportation service and eliminating jobs.
INCORRECT
This statement is not consistent with our pre-thinking. Cutting retirement benefits must be a feasible way to resolve the issue for the conclusion to hold. Hence it is not the correct choice.

(C) Other things being equal, it is in the transportation employees' interest to have exceptionally generous retirement benefits.
INCORRECT
This assumption is not necessary for the editorial’s conclusion to hold. Hence it is an incorrect choice.

(D) Cutting the retirement benefits would help resolve the agency's budget shortfall.
CORRECT
This is in line with our pre-thought assumption-1 and hence the correct choice.

(E) The transportation employees' union will not accept cuts in retirement benefits if doing so will not allow more transportation employees to keep their jobs.
INCORRECT
This statement is not regarding the context of the conclusion, i.e. whether accepting the cut in retirement benefits is in the best interest of the employees or not. Hence this option is out of the scope of the argument. Therefore, it is an incorrect choice.
Manager
Manager
Joined: 15 May 2017
Status:Discipline & Consistency always beats talent
Posts: 146
Own Kudos [?]: 124 [4]
Given Kudos: 132
Location: United States (CA)
GPA: 3.59
WE:Sales (Retail)
Send PM
Editorial: Our city's public transportation agency is facing a budget [#permalink]
3
Kudos
1
Bookmarks
Editorial: Our city's public transportation agency is facing a budget shortfall. The fastest growing part of the budget has been employee retirement benefits, which are exceptionally generous. Unless the budget shortfall is resolved, transportation service will be cut, and many transportation employees will lose their jobs. Thus, it would be in the employees' best interest for their union to accept cuts in retirement benefits.


Budget Shortfall
% of the budget comes from RETIREMENT BENEFITS
Transportation Services will not be cut or many transportation employees will not lose their jobs -> the budget is resolved
Conclusion: Accepting the cut in retirement benefits is in employee's best interest. (WHY?)

Ask: Why accepting the cut in retirement benefits is in their interest? Aren't they losing the benefits? Isn't it supposed to be bad? If my manager offer to cut my benefit, why should I accept that? That's stupid.

But, wait a minute, It is certain that either transportation service will be cut or my job will be cut if the budget short fall is not resolved. Is there anyway to save my job then :(? Hmmmmm.. It seems like there is only one option available as my manager offers: CUT my benefits. Would I still lose my job if my benefits are cut? If so, then CUTTING my benefits does not resolve the budget shortfall then. Hence, It's not my best interest to accept the CUT since either ways i am going to lose my job.

Therefore, CUTTING my benefits better works out. Otherwise, It's no where close to be benefit me ;(.

Which of the following is an assumption the editorial's argument requires?

Quote:
(A) The transportation employees' union should not accept cuts in retirement benefits if doing so would not be in the employees' best interest.


Well! if cutting my benefits works out, I will be able to keep my job. Then I should accept the cut. If cutting my benefits does not work out, I would not accept the cut since I will lose my job. But I don't know whether it is going to work out or not. (A) is out
Quote:
(B) The only feasible way for the agency to resolve the budget shortfall would involve cutting transportation service and eliminating jobs.

Wait! cutting my job is part of the only feasible way, then how the heck could it be my best interest to accept cutting my retirement benefits? So even after I accept the cut, I still lose my job? I would rather keep my benefits till my last day of work! (B) is out.
Quote:
(C) Other things being equal, it is in the transportation employees' interest to have exceptionally generous retirement benefits.

Great! My generous retirement benefits actually benefit me a lot. But they are considering cutting it:(... Also, they don't know whether the plan will work. (C) is irrelevant. Out.
Quote:
(D) Cutting the retirement benefits would help resolve the agency's budget shortfall.

Okay! The plan is going to work, I will be able to keep my job. :) Hang on to (D)
Quote:
(E) The transportation employees' union will not accept cuts in retirement benefits if doing so will not allow more transportation employees to keep their jobs.

Allowing "more employees to keep their jobs". Does that include me and my friends? Not sure. So the plan might or might nor work for us. (E) is out.

Only D is left. D is the correct answer.
GMAT Club Verbal Expert
Joined: 13 Aug 2009
Status: GMAT/GRE/LSAT tutors
Posts: 6917
Own Kudos [?]: 63648 [4]
Given Kudos: 1773
Location: United States (CO)
GMAT 1: 780 Q51 V46
GMAT 2: 800 Q51 V51
GRE 1: Q170 V170

GRE 2: Q170 V170
Send PM
Re: Editorial: Our city's public transportation agency is facing a budget [#permalink]
3
Kudos
1
Bookmarks
Expert Reply
samgyupsal wrote:
Hi experts, MartyTargetTestPrep GMATNinja AjiteshArun

The answer is definitely D, but I can't seem to understand exactly why A is incorrect. Is it because the choice flips employees' interest and the union's acceptance and negates both? Therefore, it talks about NOT accepting cuts in benefits, therefore straying away from the point of the argument - i.e., what the union should accept.

I originally ruled out this option because it states "the transportation employees' union SHOULD not accept cuts...if doing so...." Why should something that the union should or should not be assumed?

The question asks for an assumption REQUIRED by the argument. So, as you look through the answer choices, you're looking for something that MUST be true in order for the author's argument to hold water.

The author concludes that "it would be in the employees' best interest for their union to accept cuts in retirement benefits."

The reasoning behind this conclusion is:
  • There is a budget shortfall
  • The fastest growing piece of the budget is retirement benefits
  • If the budget shortfall is not resolved, a bunch of transportation employees will lose their jobs

So, which answer choice MUST be true in order to conclude that ""it would be in the employees' best interest for their union to accept cuts in retirement benefits"?

Here's (A):
Quote:
(A) The transportation employees' union should not accept cuts in retirement benefits if doing so would not be in the employees' best interest.

This doesn't absolutely HAVE to be true in order for the author to reach his/her conclusion. Perhaps there IS a circumstance under which it makes sense for the union to accept retirement cuts, even if it's not in the employees' best interests.

Maybe, for example, the cut in retirement benefits would directly fund some other really important project (building a new hospital or something). In this case, it's possible that the transportation union SHOULD support the cut in benefits even though it ISN'T in the best interests of the employees.

That doesn't impact the author's conclusion that it WOULD be in the best interest of the employees to accept THESE particular cuts in retirement benefits.

Because the argument doesn't depend on (A), (A) is not the correct answer.

Compare that to (D):
Quote:
(D) Cutting the retirement benefits would help resolve the agency's budget shortfall.

From the passage, we know that if the budget shortfall is not resolved, many transportation employees will lose their jobs. From this, the author concludes that it would be in the employees' best interests for their union to accept cuts in retirement benefits.

(D) provides a necessary link between the author's conclusion and the evidence of the passage -- in order to conclude that it is in the employees' best interest to accept the cuts, we NEED to know that the cuts will actually help resolve the budget shortfall.

Because (D) MUST be true in order for the author's argument to hold, (D) is the correct answer.

I hope that helps!
Senior Manager
Senior Manager
Joined: 29 Oct 2015
Posts: 468
Own Kudos [?]: 254 [3]
Given Kudos: 293
Send PM
Re: Editorial: Our city's public transportation agency is facing a budget [#permalink]
2
Kudos
1
Bookmarks
Editorial: Our city's public transportation agency is facing a budget shortfall. The fastest growing part of the budget has been employee retirement benefits, which are exceptionally generous. Unless the budget shortfall is resolved, transportation service will be cut, and many transportation employees will lose their jobs. Thus, it would be in the employees' best interest for their union to accept cuts in retirement benefits.

Which of the following is an assumption the editorial's argument requires?

A. The transportation employees' union should not accept cuts in retirement benefits if doing so would not be in the employees' best interest.
B. The only feasible way for the agency to resolve the budget shortfall would involve cutting transportation service and eliminating jobs.
C. Other things being equal, it is in the transportation employees' interest to have exceptionally generous retirement benefits.
D. Cutting the retirement benefits would help resolve the agency's budget shortfall.
E. The transportation employees' union will not accept cuts in retirement benefits if doing so will not allow more transportation employees to keep their jobs.
------------------------------------------------------------------------------------------------------------------------------------------------------------------------------
The conclusion is :- " it would be in the employees' best interest for their union to accept cuts in retirement benefits."
Only option B and D are close.
Option B if negated , "The only feasible way for the agency to resolve the budget shortfall would NOT involve cutting transportation service and eliminating jobs. "
But "the only feasible way for the agency to resolve the budget shortfall" may involve "cutting retirement benefits".
So then the union should accept cuts in retirement benefits.
The conclusion follows even after negation. We can say that "it would be in the employees' best interest for their union to accept cuts in retirement benefits."
So option B is NOT the assumption.


Option D if negated , "Cutting the retirement benefits would NOT help resolve the agency's budget shortfall."
Then the union should NOT accept cuts in retirement benefits.
The conclusion can not follow after negation. it would NOT be in the employees' best interest for their union to accept cuts in retirement benefits.
Option D is the assumption.

Option A , C , E are clearly irrelevant.

The correct answer is option D.
Please give me KUDO s if you liked my explanation.

VeritasKarishma GMATNinja generis
Manager
Manager
Joined: 01 Dec 2018
Posts: 147
Own Kudos [?]: 144 [3]
Given Kudos: 333
Concentration: Entrepreneurship, Finance
Schools: HBS '21 ISB'22
GPA: 4
WE:Other (Retail Banking)
Send PM
Re: Editorial: Our city's public transportation agency is facing a budget [#permalink]
3
Kudos
iamsiddharthkapoor wrote:
I selected option B because if the statement in it is not true, then that means there is/are other ways to resolve the deficit rather than to cut transportation services, causing loss in jobs, so in such a case there's no need for the employee union to accept cuts in retirement benefits.

Could anyone please tell me where's the gap in my reasoning?


Hi,
I found the difference between B and D .

METHOD 1 -First keep in mind below 2 things while negating the options you choose as final answer
1)the correct assumption answer must support the conclusion (preferably in direct way ).
2)When you negate the sentence make sure what you can negate and what you can't (facts can't be negated)
3)the negative statement should have strong effect on conclusion (so strong that the conclusion shouldn't be able to withstand any longer).

METHOD 2 - If you want you may rethink before having a look at options .
STEP 1 - negate the conclusion and analyse the reasoning used by author to come on that conclusion.
STEP 2 - think under what scenario the negated conclusion will be true
STEP 3 - the opposite of those scenarios will be assumptions we want.

Both the above methods are reverse order of each other .

QUESTION -
Editorial: Our city's public transportation agency is facing a budget shortfall. The fastest growing part of the budget has been employee retirement benefits, which are exceptionally generous. Unless the budget shortfall is resolved, transportation service will be cut, and many transportation employees will lose their jobs. Thus, it would be in the employees' best interest for their union to accept cuts in retirement benefits.

the conclusion strictly says - it would be in the employees' best interest for their union to accept cuts in retirement benefits .
OUR NOTES ON CONCLUSION
1)by reaching this goal they can address the budget shortfall .
2)their main focus to manage budget shortfall is to cut retirement benefits because even before conclusion their eyes are on employee retirement benefits. (read second line - The fastest growing part of the budget has been employee retirement benefits, which are exceptionally generous.)

OPTIONS (B V/S D)
B) The only feasible way for the agency to resolve the budget shortfall would involve cutting transportation service and eliminating jobs.
>It says would involve which means there are other ways as well and this assumption does not support conclusion 100%. conclusion clearly says that CUTS IN RETIREMENT BENEFITS WOULD RESOLVE SHORTFALL when did the eliminating jobs come in picture ? these are simply part of what will happen unless we meet shortfall .The conclusion is not based on this at all .
So when the positive part is not supporting conclusion there is not point in negating this option .
But still if you want you can check
NEGATED - The only feasible way for the agency to resolve the budget shortfall would NOT involve cutting transportation service and eliminating jobs. - yeah so If it would not involve then that means we should cut employee retirement benefits to cover SHORTFALLS . this in noway weakens or threatens the conclusion .

(D) Cutting the retirement benefits would help resolve the agency's budget shortfall.
THIS IS EXACTLY WHAT THE CONCLUSION IS TRYING TO SAY AND THIS ASSUMPTION SUPPORTS THE PASSAGE.

NEGATED - Cutting the retirement benefits would NOT help resolve the agency's budget shortfall.
Okay now after negating opt D the conclusion to cut retirement benefits is no longer valid and HENCE CONCLUSION BREAKS .

I hope I explained it well.
Target Test Prep Representative
Joined: 24 Nov 2014
Status:Chief Curriculum and Content Architect
Affiliations: Target Test Prep
Posts: 3480
Own Kudos [?]: 5134 [3]
Given Kudos: 1431
GMAT 1: 800 Q51 V51
Send PM
Re: Editorial: Our city's public transportation agency is facing a budget [#permalink]
3
Kudos
Expert Reply
samgyupsal wrote:
Hi experts

The answer is definitely D, but I can't seem to understand exactly why A is incorrect. Is it because the choice flips employees' interest and the union's acceptance and negates both? Therefore, it talks about NOT accepting cuts in benefits, therefore straying away from the point of the argument - i.e., what the union should accept.

I originally ruled out this option because it states "the transportation employees' union SHOULD not accept cuts...if doing so...." Why should something that the union should or should not be assumed?

You keep getting fooled or confused by the same thing, choices that go in a direction somehow opposite the direction the correct answer has to go in.

The argument is about what the unions SHOULD do since something is the case. (A) goes in the opposite direction, supporting what the unions SHOULD NOT do.

To master CR, you have to address this weakness in your analytical skills.
Tutor
Joined: 16 Oct 2010
Posts: 14816
Own Kudos [?]: 64880 [3]
Given Kudos: 426
Location: Pune, India
Send PM
Re: Editorial: Our city's public transportation agency is facing a budget [#permalink]
2
Kudos
1
Bookmarks
Expert Reply
Aditya9270 wrote:
aggvipul wrote:
TheRzS wrote:
Could an expert please weigh in?

Q1.
A. The transportation employees' union should not accept cuts in retirement benefits if doing so would not be in the employees' best interest.
E. The transportation employees' union will not accept cuts in retirement benefits if doing so will not allow more transportation employees to keep their jobs.

In A and E, does "doing so" stand for ACCEPTING or NOT ACCEPTING?

Q2.
Could someone please try negating A and E? Negating them seems to hurt the argument.

Thanks and regards

TheRzS Q1 -"doing so" here is indicating "Accepting"
Q2- Negating the option A. rather warps the information or the intent of what is being said, try this
A1. The transportation employees' union should accept cuts in retirement benefits if doing so would not be in the employees' best interest- it rather now intents to say that employees should accept loss
A2. The transportation employees' union should not accept cuts in retirement benefits if doing so would be in the employees' best interest- same here, it intents to say that employees should accept loss
Therefore, it does not hurt the argument rather warps the gist of question stem
would be happy to help in case anything is still unclear


GMATNinja
generis
VeritasKarishma

Dear Experts,

Please suggest what negated statements would option A and E have,
since there are two verbs -('should not accept' and 'would not be in favour')

do we need to negate both or any one of them, and on what grounds should we negate the verb out of the two

thanks


First note that all options need not be negated. Learn to identify the irrelevant options right away to save time.

When negating a conditional statement, we negate the main clause.

"If A, then B" negation is "If A, then not B"

Also note that "negation" is not "opposite". It is the complimentary set.
So negation of "B must happen" is NOT "B must not happen."
It would be "B can happen or may not happen". Basically, we don't have strong feelings for whether it should or should not happen. Either is acceptable.

Negation of
If accepting cuts would not be in the employees' best interest, the union should not accept cuts.
will be
If accepting cuts would not be in the employees' best interest, the union may or may not accept cuts.
Tutor
Joined: 16 Oct 2010
Posts: 14816
Own Kudos [?]: 64880 [3]
Given Kudos: 426
Location: Pune, India
Send PM
Re: Editorial: Our city's public transportation agency is facing a budget [#permalink]
3
Kudos
Expert Reply
jabhatta2 wrote:
Hi VeritasKarishma

Quote:
Option B) The only feasible way for the agency to resolve the budget shortfall would involve cutting transportation service and eliminating jobs.


When one negates B, negated B becomes

The only feasible way for the agency to resolve the budget shortfalf MAY OR MAY NOT involve cutting transportation service and eliminating jobs


Option (B) is tangential to our argument.

Note that as per our argument, the means to resolve budget shortfall is "retirement benefit cuts". If budget shortfall is not resolved, the result will be "cutting transportation service and eliminating jobs". Cutting transportation & eliminating jobs is not how you resolve budget shortfall. That will be the impact of budget shortfall in case it is not resolved.

Hence the argument is not assuming that "cutting transportation & eliminating jobs" is the only way to resolve budget shortfall. It is suggesting that cutting retirement benefits is a way to resolve budget shortfall.

Also, when you negate (B), you get:

(B) The only feasible way for the agency to resolve the budget shortfall would involve cutting transportation service and eliminating jobs.
Negated (B) There are other feasible ways to resolve budget shortfall too.

But we already know that. One way has already been discussed in our argument (cutting retirement benefits). It does not negate the conclusion.
Tutor
Joined: 16 Oct 2010
Posts: 14816
Own Kudos [?]: 64880 [2]
Given Kudos: 426
Location: Pune, India
Send PM
Re: Editorial: Our city's public transportation agency is facing a budget [#permalink]
1
Kudos
1
Bookmarks
Expert Reply
Prateek176 wrote:
VeritasKarishma wrote:
iamsiddharthkapoor wrote:
I selected option B because if the statement in it is not true, then that means there is/are other ways to resolve the deficit than to cut transportation services, causing loss in jobs, so in such a case there's no need for the employee union to accept cuts in retirement benefits.

Can anyone tell me what's the gap in my reasoning?


Our city's public transportation agency is facing a budget shortfall.
The fastest growing part of the budget has been employee retirement benefits, which are very high.
If budget shortfall is not resolved, transportation service will be cut, and many transportation employees will lose their jobs.

Conclusion: Thus, it would be in the employees' best interest for their union to accept cuts in retirement benefits (which will resolve budget shortfall)

Basically the argument is saying that budget shortfall is increasing while the retirement benefits are too generous. So cut down retirement benefits to take care of shortfall. Else, services will be cut and people will lose their jobs. Hence it is in people's best interest to cut retirement benefits.

We are looking for an assumption - a missing necessary premise

A. The transportation employees' union should not accept cuts in retirement benefits if doing so would not be in the employees' best interest.

Not necessary. We don't need to establish that cuts should not be accepted otherwise.

B. The only feasible way for the agency to resolve the budget shortfall would involve cutting transportation service and eliminating jobs.

Not so. Cutting retirement benefits is a way to resolve budget shortfall and is already discussed in the argument. There could be others. The argument tells us that if the budget shortfall is not resolved, then it will lead to cutting services and eliminating jobs.

C. Other things being equal, it is in the transportation employees' interest to have exceptionally generous retirement benefits.

No. The argument implies that retaining jobs is in the employees' best interest.

D. Cutting the retirement benefits would help resolve the agency's budget shortfall.

Correct. The argument assumes that cutting retirement benefits will fix the problem. That budget shortfall based job cuts will not happen in that case.

E. The transportation employees' union will not accept cuts in retirement benefits if doing so will not allow more transportation employees to keep their jobs.

No, we don't know the conditions under which retirement benefit cuts may or may not be accepted. What the union will do, we don't know. The argument only talks about what it should do.

Answer (D)


VeritasKarishma

Can you please elaborate your reasoning for eliminating A. I didn't understand your earlier explanation


The argument says that retirement benefit cuts are in people's best interest.
Hence, the union should accept the cuts.

Option (A) The transportation employees' union should not accept cuts in retirement benefits if doing so would not be in the employees' best interest.

Basically, this option says that if cuts were not in employees' best interest, union should not accept them.

The point is that it is irrelevant to our argument. If something is not in the employees best interest, how the union should react is not known to us. It doesn't pertain to our argument at all. The argument only talks about what happens when something is in employees best interest.
So option (A) is irrelevant.
Founder
Founder
Joined: 04 Dec 2002
Posts: 37296
Own Kudos [?]: 72857 [2]
Given Kudos: 18857
Location: United States (WA)
GMAT 1: 750 Q49 V42
GPA: 3
Send PM
Editorial: Our city's public transportation agency is facing a budget [#permalink]
2
Kudos
Expert Reply
Did you notice a nice little Trap in B?
(I did not and took me 10 mins to figure it out)

Here is the 1-sentence explanation why B is wrong: B says there is only one way of resolving budget shortfall which is service+job cuts. I missed this while reading. The stem presents these as alternatives to each other but B presents them as a single solution. Did you catch it?

(B) The only feasible way for the agency to resolve the budget shortfall would involve cutting transportation service and eliminating jobs.

If (B) said the only ways (plural), it would have been harder to eliminate. D is still more GMAT-like where it tackles the main rather than peripheral argument.

This is why every word on CR matters. If you missed this question and feel as silly as I do, make sure you fix your attention on the next question and notice the plural vs. singular. It is a nice little trap.
VP
VP
Joined: 14 Feb 2017
Posts: 1115
Own Kudos [?]: 2162 [1]
Given Kudos: 368
Location: Australia
Concentration: Technology, Strategy
GMAT 1: 560 Q41 V26
GMAT 2: 550 Q43 V23
GMAT 3: 650 Q47 V33
GMAT 4: 650 Q44 V36
GMAT 5: 600 Q38 V35
GMAT 6: 710 Q47 V41
WE:Management Consulting (Consulting)
Send PM
Re: Editorial: Our city's public transportation agency is facing a budget [#permalink]
1
Kudos
The answer choices makes it seem as if the argument is what is best for the agency not for the employees.

The official explanation reasoning states: "If cutting the employees' retirement benefits would not be sufficient to resolve the budget shortfall, then it may well not be in employees' best interest".

Either way, cutting benefits is never in an employees best interest, but now I see that if benefits are cut and the shortfall still isn't resolved then the Agency may still very well cut jobs, so this is why answer (C) is correct!
Manager
Manager
Joined: 12 Mar 2017
Posts: 185
Own Kudos [?]: 88 [1]
Given Kudos: 87
Location: India
Concentration: Strategy, General Management
GMAT 1: 700 Q49 V37
GPA: 4
Send PM
Re: Editorial: Our city's public transportation agency is facing a budget [#permalink]
1
Kudos
VeritasKarishma wrote:
iamsiddharthkapoor wrote:
I selected option B because if the statement in it is not true, then that means there is/are other ways to resolve the deficit than to cut transportation services, causing loss in jobs, so in such a case there's no need for the employee union to accept cuts in retirement benefits.

Can anyone tell me what's the gap in my reasoning?


Our city's public transportation agency is facing a budget shortfall.
The fastest growing part of the budget has been employee retirement benefits, which are very high.
If budget shortfall is not resolved, transportation service will be cut, and many transportation employees will lose their jobs.

Conclusion: Thus, it would be in the employees' best interest for their union to accept cuts in retirement benefits (which will resolve budget shortfall)

Basically the argument is saying that budget shortfall is increasing while the retirement benefits are too generous. So cut down retirement benefits to take care of shortfall. Else, services will be cut and people will lose their jobs. Hence it is in people's best interest to cut retirement benefits.

We are looking for an assumption - a missing necessary premise

A. The transportation employees' union should not accept cuts in retirement benefits if doing so would not be in the employees' best interest.

Not necessary. We don't need to establish that cuts should not be accepted otherwise.

B. The only feasible way for the agency to resolve the budget shortfall would involve cutting transportation service and eliminating jobs.

Not so. Cutting retirement benefits is a way to resolve budget shortfall and is already discussed in the argument. There could be others. The argument tells us that if the budget shortfall is not resolved, then it will lead to cutting services and eliminating jobs.

C. Other things being equal, it is in the transportation employees' interest to have exceptionally generous retirement benefits.

No. The argument implies that retaining jobs is in the employees' best interest.

D. Cutting the retirement benefits would help resolve the agency's budget shortfall.

Correct. The argument assumes that cutting retirement benefits will fix the problem. That budget shortfall based job cuts will not happen in that case.

E. The transportation employees' union will not accept cuts in retirement benefits if doing so will not allow more transportation employees to keep their jobs.

No, we don't know the conditions under which retirement benefit cuts may or may not be accepted. What the union will do, we don't know. The argument only talks about what it should do.

Answer (D)


VeritasKarishma

Can you please elaborate your reasoning for eliminating A. I didn't understand your earlier explanation
Tutor
Joined: 16 Oct 2010
Posts: 14816
Own Kudos [?]: 64880 [1]
Given Kudos: 426
Location: Pune, India
Send PM
Re: Editorial: Our city's public transportation agency is facing a budget [#permalink]
1
Kudos
Expert Reply
jabhatta2 wrote:
Hi GMATNinja - how do you re-phrase (A) and (E) ?

The double negatives (marked in red) coupled with the "IF STATEMENT" [underline] is very hard to wrap your head around

Quote:
(A) The transportation employees' union should not accept cuts in retirement benefits if doing so would not be in the employees' best interest.


Assume
X = best interests
Y = accept cuts

Option (A): The transportation employees' union should not accept cuts in retirement benefits if doing so would not be in the employees' best interest.

The structure of (A) is: If not X, not Y

It implies:
Not X -> Not Y
and
Y -> X

This is the same as

Only if X, then Y

which implies:
Y-> X
and
Not X -> Not Y
GMAT Club Bot
Re: Editorial: Our city's public transportation agency is facing a budget [#permalink]
 1   2   3   4   
Moderators:
GMAT Club Verbal Expert
6917 posts
GMAT Club Verbal Expert
238 posts
CR Forum Moderator
832 posts

Powered by phpBB © phpBB Group | Emoji artwork provided by EmojiOne